Spørsmål:
Når den sentrale grensesetningen og loven om store tall er uenige
Alecos Papadopoulos
2018-06-25 12:02:15 UTC
view on stackexchange narkive permalink

Dette er egentlig en replikering av et spørsmål jeg fant over på math.se, som ikke fikk svarene jeg håpet på.

La $ \ {X_i \} _ {i \ in \ mathbb {N}} $ være en sekvens av uavhengige, identisk fordelte tilfeldige variabler, med $ \ mathbb {E} [X_i] = 1 $ og $ \ mathbb {V} [X_i] = 1 $.

Vurder evalueringen av

$$ \ lim_ {n \ to \ infty} \ mathbb {P} \ left (\ frac {1} {\ sqrt {n}} \ sum_ {i = 1} ^ n X_i \ leq \ sqrt {n} \ right) $$

Dette uttrykket må manipuleres ettersom begge sider av ulikhetshendelsen, som det er, har en tendens til uendelig.

A) PRØV SUBTRAKSJON

Før du vurderer den begrensende setningen, trekker du $ \ sqrt {n} $ fra begge sider:

$$ \ lim_ {n \ to \ infty} \ mathbb {P} \ left (\ frac {1} {\ sqrt {n}} \ sum_ {i = 1} ^ n X_i - \ sqrt {n} \ leq \ sqrt {n} - \ sqrt {n} \ right) = \ lim_ {n \ to \ infty} \ mathbb {P} \ left (\ frac {1} {\ sqrt {n}} \ sum_ {i = 1} ^ n (X_i - 1) \ leq 0 \ høyre) \\ = \ Phi (0) = \ frac {1} {2} $$

den siste likheten ved CLT, der $ \ Phi () $ er standard normalfordelingsfunksjon.

B) PRØV MULTIPLIKASJON

Multipliser begge sider med $ 1 / \ sqrt {n} $ $$ \ lim_ {n \ to \ infty} \ mathbb {P} \ left (\ frac {1} {\ sqrt {n}} \ cdot \ frac {1} {\ sqrt {n}} \ sum_ {i = 1} ^ n X_i \ leq \ frac {1} {\ sqrt {n}} \ cdot \ sqrt {n} \ right) = \ lim_ {n \ to \ infty} \ mathbb {P} \ left (\ frac { 1} {n} \ sum_ {i = 1} ^ n X_i \ leq 1 \ høyre) $$

$$ = \ lim_ {n \ to \ infty} \ mathbb {P} \ left (\ bar X_n \ leq 1 \ right) = \ lim_ {n \ to \ infty} F _ {\ bar X_n} (1 ) = 1 $$

hvor $ F _ {\ bar X_n} () $ er fordelingsfunksjonen til prøven betyr $ \ bar X_n $, som av LLN konvergerer i sannsynlighet (og så også i distribusjon) til konstant $ 1 $, derav siste likestilling.

S Så vi får motstridende resultater. Whvilken er den rette? Og hvorfor den andre er galt?

En lenke til math.se-spørsmålet vil kanskje være nyttig
@JuhoKokkala Visst, her er det, https://math.stackexchange.com/q/2830304/87400 Bare ignorere OPs feil der.
Jeg tror problemet er i den andre uttalelsen som påkaller LLN
@Glen_b Simulering av den opprinnelige uttalelsen (sum delt på kvadratrot), antyder imidlertid det motsatte.
Jeg fulgte deg helt opp til den endelige likestillingen.Det er helt klart feil, fordi vi forventer at $ \ mathbb {P} (\ bar X_n \ le 1) $ tilnærmet $ 1/2 dollar for store $ n $, og grensen bør derfor ikke være lik $ 1. $ Hva er den tiltenkte begrunnelsen forden?Det er ikke uttalelsen om noen versjon av en lov om store tall som jeg kjenner.
@whuber Angivelig antas det at all sannsynlighet for prøven betyr konsentrerer seg til verdien $ 1 $.Hvis dette er galt, tror jeg det er viktig at feilen blir beskrevet i et svar, det er formålet med dette spørsmålet.
Jeg er forvirret av det fordi det ikke er noen uttalelse om en lov med stort antall som innebærer din konklusjon, og det er uklart hvilken begrunnelse du prøver å påberope for å rettferdiggjøre det.Du kan avklare spørsmålet ditt ved å (a) sitere teoremet du tror du bruker i detalj, og (b) forklare hvorfor du tror det gjelder i denne situasjonen.
@ whuber Men "argumentet" er allerede skrevet: utvalget betyr konvergerer i sannsynlighet og så også i distribusjon til en konstant.Hvorfor mer enn det er nødvendig for at noen skal vise at det er galt?
Jeg tror ikke $ \ Phi (0) $ trinnet gjelder.Det forrige trinnet virker udefinert, ikke 0. Du argumenterer effektivt for at $ P (\ infty \ cdot 0 \ leqslant 0) \ equiv \ Phi (0) $ som det ikke er.Derfor er subtraksjonsvarianten ikke nyttig.
Jeg er også uenig i den andre konklusjonen din.Etter mitt syn er $$ \ lim_ {n \ to \ infty} P (\ bar X_n \ leq 1) = P (\ mathbb {E} [X] \ leq 1) $$, og ifølge CLT, $ \ mathbb {E} [X] $ er gaussisk med $ \ mu = 1 $, derfor vil sannsynligheten være lik 1/2.
Når det er sagt, hvis du formulerer subtraksjonsvarianten på en måte som uttrykker $ \ mathbb {E} [X] $, så får du den samme konklusjonen på 1/2, så de to er konsistente i den forstand.
Tasos Papastylianou, hvis du følger logikken din, vil du oppnå $ P (1 \ leq 1) = 1 $ og ikke 1/2. Poenget er at den siste likestillingen fra Alecos Papadopoulos er feil.Vi har $ P (\ overline {X} _n \ leq x) \ til 1 $ for $ x> 0 $, og $ P (\ overline {X} _n \ leq x) \ til 0 $ for $ x <0 $mens vi for $ x = 0 $ (diskontinuitetspunkt for fordelingen av $ Y \ ekv. 1 $) har $ P (\ overline {X} _n \ leq x) \ til 0,5 $. For et inutivt eksempel la $ X_1, \ dots, X_n $ iid $ N (1,1) $ slik at $ \ overline {X} _n \ sim N (1,1 / n) $ nøyaktig;da holder det tydelig at $ P (\ overline {X} _n \ leq 1) \ til 0,5 $.
@chRrr får du bare $ P (1 \ leqslant 1) $ hvis du tar ut $ E [X] $ som om den ikke var distribuert og _nødvendigvis_ lik $ 1 $.Som er tilfelle for en sterk LLN, slik jeg forstår det, og i en CLT tilsvarer det å si at variansen på $ E [X] $ har kollapset til 0, noe som gjør distribusjonen til et delta snarere enn en ekte gaussisk.Så slik jeg forstår det, avhenger den riktige tolkningen av alt det ovennevnte av om man angriper dette på en sterk / analytisk måte, eller en svak / numerisk.
Jeg mistenker at hvis Alecos prøver å bekrefte dette numerisk for n = "et veldig stort antall effektivt uendelig", får han p = 1/2 for enhver tilnærming som tester nøyaktig hvor mye av fordelingen er <= 1, og p = 1for enhver tilnærming som tester om hele fordelingen ligger innenfor en veldig liten terskel til venstre og høyre for 1.
E (X) er ikke en tilfeldig variabel.(og jeg gjorde en feil i kommentaren min. her er sakene x <1, x = 1, x> 1 i stedet for x <0, x = 0, x> 0).
@alecos uttalelsen min var i samsvar med mine etterfølgende simuleringer (jeg sjekket før jeg kommenterte);argumentet basert på LLN var ikke i samsvar med simuleringer
Alecos, min bekymring er ikke om det siste trinnet er galt: det gjelder * dine grunner til å gjøre det. * Er det ikke det spørsmålet handler om?Jeg har fremdeles ikke lest noe fra deg med de grunnene, og jeg vil nøle med å gjette hva de kan være.Selv om du refererer til en "LLN", tror jeg at løsningen på problemet ditt sannsynligvis vil ligge i å beskrive nøyaktig hva du forstår "LLN" å hevde.
@whuber Det er velkjent hva LLN hevder _ verbalt_: at, i vårt tilfelle, "$ X_n $ konvergerer sannsynlig til $ 1 $".Deretter lærer vi at "konvergens i sannsynlighet" innebærer "konvergens i distribusjon".Det er denne kjeden av implikasjoner, påført raskt, som fører til det tilsynelatende korrekte og så motstridende resultatet av tilnærming B).Målet her er for et svar som viser hvordan, hvor og hvorfor feilen i denne 2. tilnærmingen mislykkes.Dette ville være verdifullt, fordi det er sannsynlig at den daglige brukeren av grunnleggende begrensningsteori kan prøve denne 2. tilnærmingen for å evaluere den opprinnelige grensen.
@Glen_b beklager teaseren, jeg var _sikker_ (ikke bare nesten sikkert) sikker på at du har gjort simuleringene dine før du kommenterer.Mine ble ikke utvidet nok og pålitelig veiledende.
Det ville være fint om du redigerte innlegget for å inkludere de tidligere kommentarene.BTW, jeg er ikke så sikker på påstanden din om at noen (uoppgitt til akkurat nå) "verbal" beretning om en teorem er "velkjent."Det er lett å finne støtte for påstander om at visse * nøye matematiske uttalelser * om konvergenslover er "velkjente" - eller i det minste mye publisert og sitert - men mye vanskeligere å demonstrere at (feil) omskrivninger har mye valuta.
Du samler grensen for sannsynlighetssekvensen og sannsynligheten for at begrensningsverdien er $ \ leq 1/2 $.Begge uttalelsene kan være sanne;sannsynligheten for at begrensningsverdien er $ \ leq 1/2 $ kan være lik $ 1 $ mens grensen for sannsynlighetssekvensen kan være lik $ 1/2 $.
Kan vi snakke om at sannsynligheten $ Pr (\ frac {1} {n} \ sum_ {i = 1} ^ {n} (\ bar {X} _i)) $ er godt definert for $ n \ til \ infty $?
@MartijnWeterings Gjerne.Sannsynlighetene er gitt av fordelingsfunksjonen til en konstant rv for kontinuitetspunktene, og sannsynligheten for punktet for diskontinuitet er gitt av CLT-tilnærmingen.At "samlet" disse uttrykkes av en trinnvis funksjon som ikke tilfredsstiller alle egenskapene til en distribusjonsfunksjon, betyr ikke at sannsynlighetene ikke er veldefinerte.
@MartijnWeterings Se dette svaret mitt for en uformell diskusjon om egenskapene til en CDF.https://stats.stackexchange.com/a/253121/28746.
@MartijnWeterings I vårt litt rare tilfelle oppstår 2. halvdel av sannsynlighetsmasse i forbindelse med et tall som vi ikke kan skrive - nemlig "nærmeste virkelige verdien 1 fra høyre".Vi kan ikke skrive det - men det eksisterer.
Jeg er fortsatt litt forvirret fordi det er så mange beskrivelser der ute.En populær visning er https://en.wikipedia.org/wiki/Normal_distribution#Zero-variance_limit som er relatert til spørsmålet uten å tenke på LLN (vi kan like gjerne direkte vurdere $ \ bar {X} _n \ sim N (1, \ frac {1} {n}) $).Ifølge den visningen har du $ lim_ {n \ to \ infty} P (\ bar {X} _n - 1 \ leq 0) = 1 $ mens du bruker CLT-uttrykket, får du $ P (\ sqrt {n} (\ bar{X} _n - 1) \ leq 0) = 0,5 $.Dette får meg til å lure på om $ \ bar {X} _n - 1 \ leq 0 $ er en veldefinert hendelse for $ n \ til \ infty $
Fem svar:
Alex R.
2018-06-25 13:03:11 UTC
view on stackexchange narkive permalink

Feilen her er sannsynlig i følgende faktum: konvergens i distribusjon antar implisitt at $ F_n (x) $ konvergerer til $ F (x) $ ved kontinuitetspunkter på $ F (x)$.Ettersom grensefordelingen har en konstant tilfeldig variabel, har den en hoppdiskontinuitet på $ x = 1 $, og det er derfor feil å konkludere med at CDF konvergerer til $ F (x) = 1 $.

Måten vi definerer konvergens i distribusjon utelukker ikke_ muligheten for konvergens i punktene for diskontinuitet - det krever bare ikke det.
Men hvis konvergens i distribusjon ikke krever $ F_n (1) $ for å konvergere til $ F (1) $, hva er den siste likestillingen i spørsmålet basert på?
@Juho Det er ikke basert på noe - det er kjernen i saken.* Det er ingen teorem som lar en lage den siste ligningen i spørsmålet. *
@AlecosPapadopoulos: Jeg sa aldri at det ikke utelukker muligheten.Jeg sier implisitt at du trenger å rettferdiggjøre den siste likheten utover det som er gitt deg fra konvergens i distribusjon.For eksempel hvis $ X_n $ er Bernoulli, ville det være sant.
Dilip Sarwate
2018-06-26 03:16:27 UTC
view on stackexchange narkive permalink

For iid tilfeldige variabler $ X_i $ med $ E [X_i] = \ operatorname {var} (X_i) = 1 $ definere \ begin {align} Z_n & = \ frac {1} {\ sqrt {n}} \ sum_ {i = 1} ^ n X_i, \\ Y_n & = \ frac {1} {{n}} \ sum_ {i = 1} ^ n X_i. \ end {align} Nå sier CLT at for hvert faste reelle tall $ z $, $ \ lim_ {n \ to \ infty} F_ {Z_n} (z) = \ Phi (z-1) $. OP bruker CLT for å evaluere $$ \ lim_ {n \ to \ infty} P \ left (Z_n \ leq \ frac {1} {\ sqrt {n}} \ right) = \ Phi (0) = \ frac 12. $$

Som de andre svarene samt flere av kommentarene til OPs spørsmål har påpekt, er det OPs evaluering av $ \ lim_ {n \ to \ infty} P (Y_n \ leq 1) $ som er mistenkelig. Tenk på spesielle tilfeller når iid $ X_i $ er diskrete tilfeldige variabler som tar på seg verdiene $ 0 $ og $ 2 $ med samme sannsynlighet $ \ frac 12 $. Nå kan $ \ sum_ {i = 1} ^ n X_i $ ta på seg alle til og med heltallverdier i $ [0,2n] $ og så når $ n $ er merkelig, $ \ sum_ {i = 1} ^ n X_i $ kan ikke ta på seg verdien $ n $, og dermed kan $ Y_n = \ frac 1n \ sum_ {i = 1} ^ n X_i $ ikke ta på seg verdien $ 1 $. Videre, siden fordelingen av $ Y_n $ er symmetrisk rundt $ 1 $, har vi det $ P (Y_n \ leq 1) = F_ {Y_n} (1) $ har verdien $ \ frac 12 $ når $ n $ er merkelig. Dermed sekvensen av tall $$ P (Y_1 \ leq 1), P (Y_2 \ leq 1), \ ldots, P (Y_n \ leq 1), \ ldots $$ inneholder påfølgende $$ P (Y_1 \ leq 1), P (Y_3 \ leq 1), \ ldots, P (Y_ {2k-1} \ leq 1), \ ldots $$ der alle vilkårene har verdi $ \ frac 12 $. På den annen side påfølgende $$ P (Y_2 \ leq 1), P (Y_ 4 \ leq 1), \ ldots, P (Y_ {2k} \ leq 1), \ ldots $$ er konvergerende til $ 1 $. Derfor eksisterer ikke $ \ lim_ {n \ to \ infty} P (Y_n \ leq 1) $, og påstander om konvergens på $ P (Y_n \ leq 1) $ til 1 må sees med stor mistanke.

Ben
2018-06-26 04:52:01 UTC
view on stackexchange narkive permalink

Det første resultatet ditt er det riktige.Feilen din oppstår i den andre delen, i følgende feilaktige uttalelse:

$$ \ lim_ {n \ rightarrow \ infty} F _ {\ bar {X} _n} (1) = 1. $$

Denne påstanden er usann (høyre side skal være $ \ tfrac {1} {2} $), og den følger ikke fra loven om store tall som hevdet.Den svake loven til store tall (som du påkaller) sier at:

$$ \ lim_ {n \ rightarrow \ infty} \ mathbb {P} \ Big (| \ bar {X} _n - 1 | \ leqslant \ varepsilon \ Big) = 1 \ quad \ quad \ text {for alle} \ varepsilon > 0. $$

For alle $ \ varepsilon > 0 $ betingelsen $ | \ bar {X} _n - 1 |\ leqslant \ varepsilon $ spenner over noen verdier der $ \ bar {X} _n \ leqslant 1 $ og noen verdier der $ \ bar {X} _n > 1 $.Derfor følger ikke it fra LLN at $ \ lim_ {n \ rightarrow \ infty} \ mathbb {P} (\ bar {X} _n \ leqslant 1) = 1 $.

Det (feilaktige) resultatet kommer av implikasjonen "konvergens i sannsynlighet innebærer konvergens i distribusjon".Spørsmålet angir ikke at påstanden kommer _ direkte_ fra LLN.
@AlecosPapadopoulos: Konvergens i sannsynlighet * innebærer * konvergens i distribusjon.Igjen, konvergens i distribusjon kreves bare på kontinuitetspunkter.Men kanskje du mente konvergens i sannsynlighet innebærer ikke * punktvis * konvergens av distribusjon.
@AlexR.Jeg er ikke sikker på hvor innsigelsen din ligger.Jeg tror dette problemet er dekket i mitt eget svar.
Sextus Empiricus
2018-06-25 13:31:15 UTC
view on stackexchange narkive permalink

Konvergens i sannsynlighet innebærer konvergens i distribusjon. Men ... hvilken fordeling? Hvis den begrensende fordelingen har hoppdiskontinuitet, blir grensene tvetydige (fordi flere verdier er mulige ved diskontinuiteten).

hvor $ F _ {\ bar X_n} () $ er fordelingsfunksjonen til prøven betyr $ \ bar X_n $, som ved LLN konvergerer i sannsynlighet (og så også i distribusjon) til konstant $ 1 $,

Dette er ikke riktig, og det er også enkelt å vise at det ikke kan være riktig (forskjellig fra uenigheten mellom CLT og LLN). Den begrensende fordelingen (som kan sees på som grensen for en sekvens av normalfordelte variabler) skal være:

$$ F _ {\ bar {X} _ \ infty} (x) = \ begin {cases} 0 & \ text {for} x<1 \\ 0.5& \ text {for} x = 1 \\ 1 & \ text {for} x>1 \ end {cases} $$

for denne funksjonen har du det, for en hvilken som helst $ \ epsilon>0 $ og hver $ x $, forskjellen $ | F _ {\ bar {X} _n} (x) -F _ {\ bar {X} _ \ infty} (x) | < \ epsilon $ for tilstrekkelig store $ n $. Dette mislyktes hvis $ F _ {\ bar {X} _ \ infty} (1) = 1 $ i stedet for $ F _ {\ bar {X} _ \ infty} (1) = 0,5 $


Grense for normalfordeling

Det kan være nyttig å eksplisitt skrive ut summen som brukes til å påkalle loven om store tall.

$$ \ bar {X} _n = \ frac {1} {n} \ sum_ {i = 1} ^ n X_i \ sim N (1, \ frac {1} {n}) $$

Grensen $ n \ til \ infty $ for $ \ hat {X} _n $ tilsvarer faktisk Dirac Delta-funksjonen når den er representert som grensen for normalfordelingen med variansen null.

Ved å bruke dette uttrykket er det lettere å se hva som foregår under panseret, i stedet for å bruke de ferdige lovene til CLT en LLN som tilslører begrunnelsen bak lovene.


Konvergens i sannsynlighet

Loven om store tall gir deg "konvergens i sannsynlighet"

$$ \ lim_ {n \ to \ infty} P (| \ bar {X} _n-1 | > \ epsilon) = 0 $$

med $ \ epsilon > 0 $

En tilsvarende uttalelse kan gjøres for den sentrale grensesetningen med $ \ lim_ {n \ to \ infty} P (| \ frac {1} {\ sqrt {n}} \ sum \ left (X_i-1 \ right) | > \ frac {\ epsilon} {n}) = 0 $

Det er feil å si at dette innebærer $$ \ lim_ {n \ to \ infty} P (| \ bar {X} _n-1 | >0) = 0 $$

Det er mindre hyggelig at dette spørsmålet er krysspostert så tidlig (forvirrende, men likevel interessant å se de forskjellige diskusjonene / tilnærmingene matematikk vs statistikk, så ikke så ille). svaret av Michael Hardy på matematisk stackexchange håndterer det veldig effektivt når det gjelder den sterke loven til store tall (det samme prinsippet som det aksepterte svaret fra drhab i kryssoppslaget og Dilip her). Vi er nesten sikre på at en sekvens $ \ bar {X} _1, \ bar {X} _2, \ bar {X} _3, ... \ bar {X} _n $ konvergerer til 1, men dette betyr ikke at $ \ lim_ {n \ to \ infty} P (\ bar {X} _n = 1) $ vil være lik 1 (eller det eksisterer kanskje ikke en gang som Dilip viser). Terningeksemplet i kommentarene fra Tomasz viser dette veldig pent fra en annen vinkel (i stedet for at grensen ikke eksisterer, går grensen til null). Gjennomsnittet av en terningrullsekvens vil konvergere til terningenes gjennomsnitt, men sannsynligheten for å være lik dette går til null.


Heaviside trinnfunksjon og Dirac delta-funksjon

CDF på $ \ bar {X} _n $ er følgende:

$$ F _ {\ bar {X} _n} (x) = \ frac {1} {2} \ left (1 + \ text {erf} \ frac {x-1} {\ sqrt {2 / n }} \ høyre) $$

med, hvis du vil, $ \ lim_ {n \ to \ infty} F _ {\ bar {X} _n} (1) = 0,5 $ (relatert til Heaviside-trinnfunksjonen, integrert av Dirac delta-funksjonen sett på som grensen for normalfordeling).


Jeg mener at dette synet løser ditt spørsmål angående "vis at det er galt" intuitivt, eller i det minste viser det at spørsmålet om forståelse av årsaken til denne uenigheten mellom CLT og LLN tilsvarer spørsmålet om å forstå integralenDirac delta-funksjon eller en sekvens av normalfordelinger med varians som synker til null.

Din begrensende distribusjon er faktisk ikke en distribusjon i det hele tatt.En CDF må være riktig kontinuerlig, mens den tydeligvis ikke er på $ x = 1/2 $.
Den rette kontinuiteten ser ut til å være nødvendig slik at for hver $ a $ har vi $ \ lim_ {n \ to \ infty} F_ {X} (a + \ frac {1} {n}) = F_ {X} (a) $ettersom hendelsene $ X \ leq a + \ frac {1} {n} $ er nestet, bør vi ha $$ \ lim_ {n \ to \ infty} F_ {X} (a + \ frac {1} {n}) = \lim_ {n \ to \ infty} P (X \ leq a + \ frac {1} {n}) = P (\ lim_ {n \ to \ infty} X \ leq a + \ frac {1} {n}) = P(X \ leq a) = F_ {X} (a) $$, men stemmer dette for vårt tilfelle, og hvor er fangsten?Er denne rette kontinuiteten nødvendig basert på sannsynlighetsaksiomer, eller er det bare en konvensjon slik at CDF fungerer i de vanligste tilfellene?
@Martin Weterings: Dette er nettopp der det kommer fra.Ethvert gyldig mål $ P $ må tilfredsstille disse ensformighetsresultatene.De er en konsekvens av begrensningen på $ P $ sammen med tellbar additivitet.Mer generelt er en funksjon $ F (x) $ en CDF (dvs. tilsvarer en viss fordeling $ P $ via $ F (b) -F (a) = P (a
Alecos Papadopoulos
2018-06-26 17:43:58 UTC
view on stackexchange narkive permalink

Jeg mener det nå skal være klart at "CLT-tilnærmingen" gir riktig svar.

La oss finne ut nøyaktig hvor "LLN-tilnærming" går galt.

Fra og med de endelige utsagnene, er det klart da at vi tilsvarende kan enten trekke $ \ sqrt {n} $ fra begge sider, eller multidip begge sider med $ 1 / \ sqrt {n} $. Vi får

$$ \ mathbb {P} \ left (\ frac {1} {\ sqrt {n}} \ sum_ {i = 1} ^ n X_i \ leq \ sqrt {n} \ right) = \ mathbb {P } \ left (\ frac {1} {\ sqrt {n}} \ sum_ {i = 1} ^ n (X_i-1) \ leq 0 \ right) = \ mathbb {P} \ left (\ frac {1} {n} \ sum_ {i = 1} ^ nX_i \ leq 1 \ høyre) $$

Så hvis grensen eksisterer, vil den være identisk. Innstilling av $ Z_n = \ frac {1} {\ sqrt {n}} \ sum_ {i = 1} ^ n (X_i-1) $, vi har, ved hjelp av distribusjonsfunksjoner

$$ \ mathbb {P} \ left (\ frac {1} {\ sqrt {n}} \ sum_ {i = 1} ^ n X_i \ leq \ sqrt {n} \ right) = F_ {Z_n} (0) = F _ {\ bar X_n} (1) $$

... og det er sant at $ \ lim_ {n \ to \ infty} F_ {Z_n} (0) = \ Phi (0) = 1/2 $.

Tenkningen i "LLN-tilnærmingen" går som følger: "Vi vet fra LLN at $ \ bar X_n $ konvergerer i sannsynlighet til en konstant. Og vi vet også at" konvergens i sannsynlighet innebærer konvergens i distribusjon ". , $ \ bar X_n $ konvergerer i distribusjon til en konstant ". Hittil har vi rett.
Så sier vi: "Derfor er begrensende sannsynligheter for $ \ bar X_n $ gitt av fordelingsfunksjonen til konstanten ved $ 1 $ tilfeldig variabel",

$$ F_1 (x) = \ cases {1 \; \; \; \; x \ geq 1 \\ 0 \; \; \; \; x<1} \ innebærer F_1 (1) = 1 $$

... så $ \ lim_ {n \ to \ infty} F _ {\ bar X_n} (1) = F_1 (1) = 1 $ ...

... og vi gjorde nettopp vår feil. Hvorfor? Fordi, som @ AlexR. svar bemerket, "konvergens i distribusjon" dekker bare kontinuitetspunktene til den begrensende distribusjonsfunksjonen. Og $ 1 $ er et punkt på diskontinuitet for $ F_1 $. Dette betyr at $ \ lim_ {n \ to \ infty} F _ {\ bar X_n} (1) $ kan være lik $ F_1 (1) $, men det kan ikke være uten å negere "konvergens i distribusjon til en konstant" implikasjon av LLN.

Og siden CLT-tilnærmingen vet vi hva verdien av grensen må være ($ 1/2 $). Jeg vet ikke om en måte å bevise direkte at $ \ lim_ {n \ to \ infty} F _ {\ bar X_n} (1) = 1/2 $.

D Har vi lært noe nytt?

Det gjorde jeg. LLN hevder at

$$ \ lim_ {n \ rightarrow \ infty} \ mathbb {P} \ Big (| \ bar {X} _n - 1 | \ leqslant \ varepsilon \ Big) = 1 \ quad \ quad \ text {for alle } \ varepsilon > 0 $$

$$ \ innebærer \ lim_ {n \ rightarrow \ infty} \ Big [\ mathbb {P} \ Big (1- \ varepsilon < \ bar {X} _n \ leq 1 \ Big) + \ mathbb {P} \ Big (1 < \ bar {X} _n \ leq 1+ \ varepsilon \ Big) \ Big] = 1 $$

$$ \ antyder \ lim_ {n \ rightarrow \ infty} \ Big [\ mathbb {P} \ Big (\ bar {X} _n \ leq 1 \ Big) + \ mathbb {P} \ Big (1 < \ bar {X} _n \ leq 1+ \ varepsilon \ Big) \ Big] = 1 $$

LLN sier ikke hvordan fordeles sannsynligheten i $ (1- \ varepsilon, 1+ \ varepsilon) $ -intervallet. Det jeg lærte er at i denne klassen av konvergensresultater er sannsynligheten på grensen allokert på begge sider av midtpunktet for kollapsintervallet.

Den generelle uttalelsen her er, antar

$$ X_n \ to_p \ theta, \; \; \; h (n) (X_n- \ theta) \ to_d D (0, V) $$

hvor $ D $ er noe rv med distribusjonsfunksjon $ F_D $. Så

$$ \ lim_ {n \ to \ infty} \ mathbb P [X_n \ leq \ theta] = \ lim_ {n \ to \ infty} \ mathbb P [h (n) (X_n- \ theta) \ leq 0] = F_D (0) $$

... som kanskje ikke er lik $ F _ {\ theta} (0) $ (fordelingsfunksjonen til konstant rv).

Dette er også et sterkt eksempel på at når fordelingsfunksjonen til den begrensende tilfeldige variabelen har diskontinuiteter, så kan "konvergens i distribusjon til en tilfeldig variabel" beskrive en situasjon der "den begrensende fordelingen" kan være uenig med "fordelingenav den begrensende tilfeldige variabelen "ved diskontinuitetspunktene. Strengt tatt er den begrensende fordelingen for kontinuitetspunktene den for den konstant tilfeldige variabelen.For diskontinuitetspoengene kan vi kanskje beregne den begrensende sannsynligheten, som "separate" enheter.

Perspektivet 'leksjon lært' er interessant, og dette er et godt, ikke så vanskelig eksempel for didaktisk anvendelse.Selv om jeg lurer på hva slags (direkte) praktisk anvendelse denne tenkningen om det uendelige har, fordi til slutt i praksis $ n \ neq \ infty $
@MartijnWeterings Martijn, motivasjonen her var absolutt lærerik, a) som et varsel for diskontinuiteter selv i en slik "flat" situasjon som konvergensen til en konstant, og så også generelt (de ødelegger for eksempel ensartet konvergens), og b) aresultat på hvordan sannsynlighetsmassen tildeles blir interessant når sekvensen som konvergerer i sannsynlighet til en konstant, fremdeles har en variasjon som ikke er null.
Vi kan si at CLT la oss si noe om konvergens til en begrensende normalfordelt variabel (og dermed kunne uttrykke ting som $ F (x) $), men LLN lar oss bare si at ved å øke utvalgsstørrelsen,komme nærmere det sanne gjennomsnittet, men dette sier ikke at vi med høyere sannsynlighet får 'nøyaktig lik prøvene'.LLN betyr at prøvenes gjennomsnitt kommer nærmere og nærmere en begrensningsverdi, men ikke (med større sannsynlighet) lik den.LLN sier ingenting om $ F (x) $
De opprinnelige tankene rundt LLN var faktisk motsatt (se resonnementet til Arbuthnot https://stats.stackexchange.com/questions/343268/).* "Det er synlig fra det som er blitt sagt, at med et veldig stort antall terninger ville As mye bli veldig lite ... det ville bare være en liten del av alle mulige sjanser for at det skulle skje til enhver tid som kan tildelesat like mange menn og kvinner skal fødes. "*


Denne spørsmålet ble automatisk oversatt fra engelsk.Det opprinnelige innholdet er tilgjengelig på stackexchange, som vi takker for cc by-sa 4.0-lisensen den distribueres under.
Loading...